Integrating Rational Functions with Substitution

  • Thread starter Thread starter binbagsss
  • Start date Start date
  • Tags Tags
    Integration
Click For Summary
The discussion focuses on integrating the rational function ∫(x^3/(x^2+m^2)) dx from 0 to b. A proposed solution involves integration by parts, breaking down x^3 into x^2 * x. However, the correct approach suggests using substitution with u = x^2 + m^2, simplifying the integral into more manageable parts. The final solution is expressed as b^2 + m^2 log(m^2/(b^2+m^2)). Overall, the integration technique emphasizes the effectiveness of substitution over integration by parts in this context.
binbagsss
Messages
1,291
Reaction score
12

Homework Statement



Apologies if this is obvious, maybe I'm a little out of touch

## \int\limits^b_0 \frac{x^3}{x^2+m^2} dx ##

Homework Equations

The Attempt at a Solution



I [/B]was going to go by parts breaking the ##x^3 = x^2 . x##

So that I have the logarithm

I.e :

##b^2 \frac{log (b^2 + m^2)}{2} - \int \frac{log (x^2+m^2)}{2} 2x dx ##
But the solution is :

## b^2 + m^2 log ( \frac{m^2}{b^2+m^2} ) ##( I thought that perhaps the solution could be going by parts again, but there is no reason for the boundary term to vanish )

Ta
 
Physics news on Phys.org
binbagsss said:

Homework Statement



Apologies if this is obvious, maybe I'm a little out of touch

## \int\limits^b_0 \frac{x^3}{x^2+m^2} dx ##

Homework Equations

The Attempt at a Solution



I [/B]was going to go by parts breaking the ##x^3 = x^2 . x##

So that I have the logarithm

I.e :

##b^2 \frac{log (b^2 + m^2)}{2} - \int \frac{log (x^2+m^2)}{2} 2x dx ##
But the solution is :

## b^2 + m^2 log ( \frac{m^2}{b^2+m^2} ) ##( I thought that perhaps the solution could be going by parts again, but there is no reason for the boundary term to vanish )

Ta
The second term in your answer can be integrated using substitution, with ##u = x^2 + b^2, du = 2xdx##.
 
  • Like
Likes Delta2
Write ##x^3=x(x^2+m^2)-m^2x## so the integral becomes ##\int xdx-\int \frac{m^2x}{x^2+m^2}dx##.
 
Or just use the substitution ##u=x^2+m^2## on the original integral.
 
Question: A clock's minute hand has length 4 and its hour hand has length 3. What is the distance between the tips at the moment when it is increasing most rapidly?(Putnam Exam Question) Answer: Making assumption that both the hands moves at constant angular velocities, the answer is ## \sqrt{7} .## But don't you think this assumption is somewhat doubtful and wrong?

Similar threads

Replies
3
Views
2K
  • · Replies 105 ·
4
Replies
105
Views
6K
  • · Replies 6 ·
Replies
6
Views
2K
  • · Replies 4 ·
Replies
4
Views
2K
  • · Replies 22 ·
Replies
22
Views
3K
  • · Replies 15 ·
Replies
15
Views
2K
  • · Replies 12 ·
Replies
12
Views
2K
  • · Replies 19 ·
Replies
19
Views
2K
  • · Replies 5 ·
Replies
5
Views
2K
  • · Replies 8 ·
Replies
8
Views
2K